ANALISI DELLA STABILITA :` errata corrigefornasini/04Stabilita1.pdf · un altro intero positivo...

7
Capitolo 4 ANALISI DELLA STABILIT ` A: errata corrige Esercizio 4.2.1 Se ˜ x(·)e˜ x(·)+c, c = 0, sono entrambi movimenti di un sistema lineare autonomo, allora c Xe e lo spazio dei punti di equilibrio del sistema non contiene soltanto l’origine. Viceversa, sia c Xe uno stato di equilibrio. Se ˜ x(·)` e un movimento del sistema, anche ˜ x(·)+ c lo ` e. ... Esercizio 4.4.2 Sia P una matrice simmetrica e sia V (x1,...xn)= x T P x la forma quadratica associata. Si verifichi che in ogni punto di R n la matrice hessiana di V coincide con 2P : 2 V ∂x1∂x1 ... 2 V ∂x1∂xn . . . . . . . . . 2 V ∂xn∂x1 ... ∂V 2 ∂xn∂xn =2P. ... Esempio 4.4.3 .......... si determinano la componente h-esima del gradiente ∂V ∂x h = ∂x h ( n i=1 n j=1 xi mij xj )= n i=1 xi m ih + n j=1 m hj xj = 2 n i=1 xi m ih =2x T col h (M), e quindi il gradiente ...... ... Esempio 4.4.4 [Pendolo Piano] Si consideri un pendolo piano ideale, costituto da un’asta rigida di lunghezza e massa m concentrata nell’estremit`a mobile. Indicando con R la intensit`a della 1

Transcript of ANALISI DELLA STABILITA :` errata corrigefornasini/04Stabilita1.pdf · un altro intero positivo...

Page 1: ANALISI DELLA STABILITA :` errata corrigefornasini/04Stabilita1.pdf · un altro intero positivo Mtale che kxk >Mimplica |V(x)| >N. Sia x = 0 punto di equilibrio per (??). 4 CAPITOLO

Capitolo 4

ANALISI DELLA STABILITA :

errata corrige

• Esercizio 4.2.1 Se x(·) e x(·)+c, c 6= 0, sono entrambi movimenti di un sistema lineare autonomo,allora c ∈ Xe e lo spazio dei punti di equilibrio del sistema non contiene soltanto l’origine.

Viceversa, sia c ∈ Xe uno stato di equilibrio. Se x(·) e un movimento del sistema, anche x(·) + c loe.

. . .

• Esercizio 4.4.2 Sia P una matrice simmetrica e sia V (x1, . . . xn) = xTPx la forma quadraticaassociata. Si verifichi che in ogni punto di R

n la matrice hessiana di V coincide con 2P :

∂2V

∂x1∂x1

. . .∂2V

∂x1∂xn

.... . .

...

∂2V

∂xn∂x1

. . .∂V 2

∂xn∂xn

= 2P.

. . .

Esempio 4.4.3 .......... si determinano la componente h-esima del gradiente

∂V

∂xh

=∂

∂xh

(

n∑

i=1

n∑

j=1

ximijxj) =

n∑

i=1

ximih +

n∑

j=1

mhjxj

= 2

n∑

i=1

ximih = 2xT colh(M),

e quindi il gradiente ......

. . .

Esempio 4.4.4 [Pendolo Piano] Si consideri un pendolo piano ideale, costituto da un’asta rigidadi lunghezza ℓ e massa m concentrata nell’estremita mobile. Indicando con R la intensita della

1

Page 2: ANALISI DELLA STABILITA :` errata corrigefornasini/04Stabilita1.pdf · un altro intero positivo Mtale che kxk >Mimplica |V(x)| >N. Sia x = 0 punto di equilibrio per (??). 4 CAPITOLO

2 CAPITOLO 4. ANALISI DELLA STABILITA : ERRATA CORRIGE

reazione vincolare e con g il modulo dell’accelerazione di gravita, si ricavano le equazioni del motodella massa m:

md2ξ1dt

= −R sin θ (4.25)

md2ξ2dt

= mg −R cos θ (4.26)

Poniamo ξ1 = ℓ sin θ, ξ2 = ℓ cos θ, sostituiamo tali espressioni, dopo averle derivate due volte rispettoal tempo, nelle due equazioni e moltiplichiamo (4.26) per −cosθ e (4.25) per sin θ. Sommando infinei risultati ottenuti, si perviene all’equazione non lineare del secondo ordine

d2θ

dt= −

g

ℓsin θ, (4.27)

nella quale non compare piu la reazione vincolare.Poniamo ora x1 = θ, x2 = θ; il sistema si scrivenella forma

x1 = x2 (4.28)

x2 = −g

lsin x1 (4.29)

e ha due punti di equilibrio (0, 0) e (π, 0). Pren-diamo come funzione di Lyapunov V (x1, x2) =g

l(1− cosx1) +

1

2x2

2, definita positiva nell’intorno

del primo punto di equilibrio, e calcoliamo

V (x1, x2) = [g

lsin x1]x2 + x2[−

g

lsin x1]

Essa e identicamente nulla, quindi semidefinitanegativa, e possiamo concludere che (0, 0) e puntodi equilibrio stabile.

-ξ1

?ξ2

dCCCCCCCCCCxCCCCO

CCCCO

CCCCOR

??mg

Figura 4.4.4

Dal precedente esercizio 4.4.4 segue inoltre che l’equilibrio in (0, 0) non puo essere convergente.

Per l’altro punto di equilibrio (ovviamente instabile!), la V considerata non e nulla nel punto equindi non e definita positiva, come richiesto per applicare il criterio.

Introduciamo ora l’ipotesi che il movimento del pendolo avvenga in presenza di attrito viscoso(dovuto p.es. al fluido in cui il pendolo e immerso). E facile verificare che (4.27) si modifica in

d2θ

dt= −

g

ℓsin θ −

β

m

dt, (4.30)

e il modello di stato diventa

x1 = x2 (4.31)

x2 = −g

lsin x1 −

β

mx2 (4.32)

Ricorrendo alla medesima funzione di Lyapunov, si ottiene ora

V (x1, x2) = −β

mx2

2,

che e semidefinita negativa. Si conclude ancora per la stabilita del punto di equilibrio ma, nonessendo applicabile l’esercizio 4.4.4, non si puo piu escludere la convergenza (e in realta l’equilibrioe convergente, come si vedra applicando il successivo criterio di Krasowskii).

Page 3: ANALISI DELLA STABILITA :` errata corrigefornasini/04Stabilita1.pdf · un altro intero positivo Mtale che kxk >Mimplica |V(x)| >N. Sia x = 0 punto di equilibrio per (??). 4 CAPITOLO

3

Supponiamo infine che le forze di inerzia siano trascurabili rispetto a quelle di attrito (come avvieneper un pendolo molto leggero immerso in un fluido altamente viscoso). (4.30) diventa

−mg

ℓsin θ = m

d2θ

dt+ β

dt∼ β

dt(4.33)

e si ottiene il sistema del primo ordine

θ = −mg

βlsin θ,

che ha due punti di equilibrio in θ = 0 e in θ = π. Prendiamo come funzione di LyapunovV (θ) = sin2 θ, definita positiva nell’intorno sia del primo che del secondo punto di equilibrio, ecalcoliamo

V (θ) = −dV

mg

βlsin θ = −2

mg

βlsin2 θ cos θ

Essa e nulla in entrambi i punti di equilibrio e nel loro intorno assume segno opposto a quello dicos θ. Quindi V e definita negativa in 0 e definita positiva in π.

Possiamo concludere che θ = 0 e punto di equilibrio asintoticamente stabile. Per l’equilibrio in π none applicabile il criterio considerato, mentre risulta applicabile il criterio di instabilita di Lyapunov,che studieremo piu avanti.

Esempio 4.4.5 Si consideri il sistema del secondo ordine

x1 = −x1 + p1(x1, x2)

x2 = −2x2 + p2(x1, x2)

con p1 e p2 polinomi privi del termine noto e di quelli del primo grado. Evidentemente l’origine dellecoordinate e punto di equilibrio. Per studiare la stabilita dell’equilibrio nell’origine, utilizziamo laforma quadratica definita positiva V (x1, x2) = x2

1 + x22. Si ha allora

V (x1, x2) =∂V

∂x1

(−x1 + p1(x1, x2)) +∂V

∂x2

(−2x2 + p2(x1, x2))

= 2x1(−x1 + p1(x1, x2)) + 2x2(−2x2 + p2(x1, x2))

= [x1 x2 ]

[

−2 00 −4

][

x1

x2

]

+ f3(x1, x2)

con f3 funzione polinomiale priva di termini di grado inferiore a 3. Quindi V e definita negativa(cfr Esercizio 4.4.3)) nell’intorno dell’origine e l’equilibrio nell’origine e asintoticamente stabile.

• Esercizio 4.4.5* [Generalizzazione del criterio di Lyapunov] Sia dato il sistema continuo(4.2). Se

1. esiste una funzione definita positiva V , continua nell’intornoW dell’origine e dotata di derivateprime continue soltanto in W \ {0},

2. la funzione V : W → R e continua in W \ {0} e ivi non positiva (negativa),

allora

(i) l’origine e punto di equilibrio del sistema;

(ii) l’equilibrio nell’origine e stabile (asintoticamente stabile).

• Esercizio 4.4.6* [Stabilita asintotica globale] Una funzione continua V : Rn → R e “radial-

mente illimitata” se V (x) → ∞ quando ‖x‖ → ∞. In altre parole, per ogni intero positivo N esisteun altro intero positivo M tale che ‖x‖ > M implica |V (x)| > N .

Sia x = 0 punto di equilibrio per (??).

Page 4: ANALISI DELLA STABILITA :` errata corrigefornasini/04Stabilita1.pdf · un altro intero positivo Mtale che kxk >Mimplica |V(x)| >N. Sia x = 0 punto di equilibrio per (??). 4 CAPITOLO

4 CAPITOLO 4. ANALISI DELLA STABILITA : ERRATA CORRIGE

(i) Si dimostri che se V : Rn → R+, continua con le derivate prime, e radialmente illimitata e

V (x) < 0 per x 6= 0, allora l’equilibrio nell’origine e globalmente asintoticamente stabile.

(ii) Si supponga che V : Rn → R, continua con le derivate prime, sia positiva per x 6∈ S(0, r) e

radialmente illimitata. Se V (x) ≤ 0 per x 6∈ S(0, r), tutti i movimenti del sistema sono limitati.

(iii) Si supponga che V : Rn → R+, continua con le derivate prime, sia radialmente illimitata e

che nell’insieme chiuso {x : V (x) ≥ k} si abbia V (x) < 0. Allora, qualunque sia lo stato inizialex(0), la corrispondente traiettoria del sistema al divergere di t rimane confinata nell’insieme apertocomplementare {x : V (x) < k}.

♯ Suggerimento: (i) si dimostri che l’origine e l’unico punto di minimo di V . Quindi, eventualmentesostituendo V con V − V (0), si puo ritenere V definita positiva. Dato x(0), esiste M > 0 tale chex(t) ∈ S(0,M) per ogni t ≥ 0. La convergenza a 0 del movimento si prova allora come nel teorema4.4.1.

(ii) Se ‖x(0)‖ ≥ r, indichiamo con N il valor massimo di V nella sfera chiusa di raggio ‖x(0)‖,altrimenti N rappresenta il massimo di V sulla frontiera ∂S(0, r). Si consideri poi la sfera di raggioM individuata nella definizione di funzione radialmente illimitata.

Il criterio di Krasowskii costituisce un raffinamento del criterio di Lyapunov: esso consenteinfatti di verificare la convergenza (e quindi la stabilita asintotica) dell’equilibrio in casinei quali il criterio di Lyapunov puo garantire soltanto la stabilita.

Teorema 4.4.3 [Criterio di stabilita di Krasowskii] Sia x = 0 punto di di equilibrioper il sistema (4.2). Si supponga che in un intorno W dell’origine la funzione V : W → R,continua con le derivate prime, sia definita positiva e che la funzione V : W → R siasemidefinita negativa.

Se il piu grande insieme invariante contenuto in

N := {x ∈ W : V (x) = 0}

coincide con l’origine, ovvero se ogni traiettoria con inizio in N \ {0} non e contenutainteramente in N 1, allora x = 0 e punto di equilibrio asintoticamente stabile.

Prova Fissata una sfera chiusa S(0, ǫ) ⊂ W , per il criterio di stabilita di Lyapunov esisteuna sfera S(0, δ) tale che ogni movimento x(·) del sistema con inizio in S(0, δ) rimangaconfinato in S(0, ǫ). Vogliamo provare che, nelle ipotesi assunte, se ‖x(0)‖ < δ, allora x(t)converge allo stato 0 quando t → +∞.

1questa proprieta si enuncia anche dicendo che N non contiene “traiettorie perturbate”, intendendosicome perturbata una traiettoria diversa da quella dell’equilibrio in 0.

Page 5: ANALISI DELLA STABILITA :` errata corrigefornasini/04Stabilita1.pdf · un altro intero positivo Mtale che kxk >Mimplica |V(x)| >N. Sia x = 0 punto di equilibrio per (??). 4 CAPITOLO

5

Notiamo anzitutto che la traiettoria γ con inizioin x(0), essendo contenuta in S(0, ǫ), e limitata equindi, per il teorema 4.3.2, il suo insieme limiteω(γ) e un sottoinsieme compatto e non vuoto diS(0, ǫ). La funzione V e semidefinita negativa,

mentre V e definita positiva. Quindi V (x(·)),valutata lungo la traiettoria γ, e funzione nonnegativa e non crescente di t e, passando al li-mite per t → +∞, si ottiene

limt→+∞

V (x(t)) = m ≥ 0. (4.34)

Ma ogni punto p ∈ ω(γ) e limite, per tk → +∞,di una successione {x(tk)}k∈N e il limite (4.34)non cambia se valutato lungo una successione diistanti che diverge a +∞. Per la continuita diV si ha allora

V (p) = V

(

limtk→+∞

x(tk)

)

= limtk→+∞

V (x(tk)) = limt→+∞

V (x(t)) = m

e in tutti i punti di ω(γ) la funzione V vale m.

Per l’invarianza dell’insieme limite, ogni movimento con inizio in un arbitrario punto diω(γ) si svolge in ω(γ), dove V vale m, quindi V e nullo in tutti i punti di ω(γ) e

ω(γ) ⊆ N .

Poiche ogni movimento con inizio in ω(γ) e contenuto in N , l’insieme limite non puocontenere punti diversi dall’origine, altrimenti da tali punti avrebbero inizio traiettorieperturbate interamente contenute in N . Quindi

ω(γ) = {0}

e, per il Teorema 4.3.3,

0 = limt→+∞

d(x(t), ω(γ)) = limt→+∞

d(x(t),0)

ovvero limt→+∞ x(t) = 0.

. . . . . .

• Esercizio 4.6.1 [Equazione di Lyapunov e cambiamenti di base] Sia X = M una soluzionedell’equazione F TX + XF = L. Se F = T−1FT e L = T TLT , l’equazione F TX + XF = L haT TMT come soluzione.

• Esercizio 4.6.2* [Equazione di Lyapunov con termine noto arbitrario] Si dimostri l’equivalenzadei seguenti fatti:

(i) l’equazione F TX +XF = 0 ammette soltanto la soluzione nulla;

Page 6: ANALISI DELLA STABILITA :` errata corrigefornasini/04Stabilita1.pdf · un altro intero positivo Mtale che kxk >Mimplica |V(x)| >N. Sia x = 0 punto di equilibrio per (??). 4 CAPITOLO

6 CAPITOLO 4. ANALISI DELLA STABILITA : ERRATA CORRIGE

(ii) se l’equazione F TX + XF = L, con L matrice in Rn×n, e risolubile, essa non ammette due

soluzioni distinte;

(iii) l’equazione F TX +XF = L e risolubile per ogni matrice L ∈ Rn×n.

♯ Suggerimento: si utilizzi l’equivalenza fra iniettivita e suriettivita della mappa lineare

ψ : Rn×n → R

n×n : X 7→ F TX +XF.

• Esercizio 4.6.3* [Equazione XF1 − F2X = L] Se le matrici F1 ∈ Rn1×n1 ed F2 ∈ R

n2×n2 nonhanno autovalori comuni, allora

(i) l’equazione XF1 − F2X = 0 nella matrice incognita X ∈ Rn2×n1 ammette soltanto la soluzione

nulla;

♯ Suggerimento: l’equazione e soddisfatta da X sse le matrici razionali strettamente proprieX(zIn1

−F1)−1e (zIn2

−F2)−1X coincidono. L’elemento in posizione (i, j) nella prima matrice ha

i poli compresi fra gli autovalori di F1, nella seconda ha i poli compresi fra gli autovalori di F2. Siprovi che (zIn1

− F2)−1X e quindi X sono nulle.

(ii) l’equazione XF1 − F2X = L ammette, per ogni L ∈ Rn2×n1

, una e una sola soluzione;

♯ Suggerimento: la mappa lineare ψ : Rn2×n1 → R

n2×n1 : X 7→ XF1 + F2X e iniettiva, dato cheil suo nucleo contiene solo la matrice nulla. Ma allora essa e anche suriettiva.

(iii) per ogni L ∈ Rn2×n1

, esiste una matrice triangolare a blocchi T =

[

In10

X In2

]

tale da aversi

T−1

[

F1 0L F2

]

T =

[

F1 00 F2

]

♯ Suggerimento: affinche la matrice T esista, occorre e basta che X verifichi l’equazione XF1 −F2X = L. Ma questo problema ammette soluzione per il punto precedente.

• Esercizio 4.6.4* [Risolubilita’ dell’equazione di Lyapunov con termine noto arbitrario]Si consideri l’equazione

F TX +XF = −L, (4.38)

con L generica matrice in Rn×n. Condizione necessaria e sufficiente affinche la soluzione sia unica e

che F e −F non abbiano autovalori comuni.

Si proceda attraverso i seguenti tre passi:

(i) la soluzione e unica se e solo se, al variare di X in Rn×n, la matrice F TX +XF descrive tutto

lo spazio Rn×n (cfr. l’esercizio 4.6.2);

(ii) se F e −F hanno autovalori comuni, F TX +XF non descrive tutto Rn×n;

♯ Suggerimento: se λ e −λ appartengono allo spettro di F e se v e w sono vettori non nulli percui si ha vTF T = −λvT e Fw = λw, allora vT (F TX +XF )w = 0 per ogni matrice X, assurdo!

(iii) Se F e −F non hanno autovalori comuni, esiste una soluzione di (4.38).

♯ Suggerimento: F non ha autovalori sull’asse immaginario. Con un cambiamento di base (cfr.esercizio 6.4.1) ci si riduca all’equazione

[

F T− 00 F T

+

] [

X11 X12

X21 X22

]

+

[

X11 X12

X21 X22

][

F− 00 F+

]

=

[

L11 L12

L21 L22

]

, (4.39)

dove F− e F+ sono dotate di spettro rispettivamente a parte reale negativa e a parte reale positivae dove gli spettri di F− e di −F+ sono disgiunti. Risolvere (4.39) equivale a risolvere il sistema

F T− X11 + X11F− = L11

F T− X12 − X12(−F+) = L12

(−F T+ )X21 − X21(F−) = −L21

(−F T+ )(X22) + (−X22)(F+) = −L22 (4.40)

Si verifichi che tutte le equazioni in (4.40) ammettono soluzione.

Page 7: ANALISI DELLA STABILITA :` errata corrigefornasini/04Stabilita1.pdf · un altro intero positivo Mtale che kxk >Mimplica |V(x)| >N. Sia x = 0 punto di equilibrio per (??). 4 CAPITOLO

7

• Esercizio 4.6.5* [Soluzioni simmetriche nel caso generale] Se S e un’arbitraria matricesimmetrica e se l’equazione F TX +XF = S ammette una soluzione M

(i) (M +MT )/2 e una soluzione simmetrica;

(ii) se la soluzione e unica, allora e necessariamente simmetrica;

(iii) se F =

[

0 −11 0

]

e S = 0, la soluzione non e unica, ma e unica la soluzione simmetrica.